Derivatives and continuity / Lipschitz equation

Click For Summary
The discussion centers on proving that a function with both right and left derivatives at a point is continuous at that point. The initial solution attempts to use Lipschitz conditions derived from the derivatives to establish continuity. However, questions arise regarding the application of Lagrange's theorem and whether all necessary conditions are met. Participants suggest revisiting the proof that every differentiable function is continuous as a potential alternative approach. The conversation emphasizes the need for clarity in applying mathematical theorems and conditions in proofs.
Felafel
Messages
170
Reaction score
0
Hi! I think I've managed to solve this problem, but I'd like it to be checked

Homework Statement



show that if $$f : A\subset \mathbb{R}\to \mathbb{R}$$ and has both right derivative:
$$f_{+}'(x_0),$$

and left derivative
$$f_{-}'(x_0)$$
in $$x_0\in A$$, then $$f$$
is continuos in
$$x_0.$$

The Attempt at a Solution



Let's assume $$f_{+}' > f_{-}'$$, as the derivative exists, it means it is $$< \infty$$.
Therefore, $$|f(x)-f(y)|≤f_{+}'|(x-y)|$$ is a Lipschitz equation, and for
$$|f(x)-f(x_0)|≤f_{+}'|(x-x_0)|$$
it is a lipschitz equation in x_0.
Thus, for the lipschitz equation properties, the function is continuos in $$x_0$$
 
Last edited:
Physics news on Phys.org
Felafel said:
Therefore, $$|f(x)-f(y)|≤f_{+}'|(x-y)|$$ is a Lipschitz equation, and for
$$|f(x)-f(x_0)|≤f_{+}'|(x-x_0)|$$

How exactly did you obtain these inequalitities??

Also, you should type # instead of $. Typing

Code:
$$ ... $$

automatically places everything on a separate line. The command

Code:
## ... ##

does not do this.
 
micromass said:
How exactly did you obtain these inequalitities??

##|f(x)−f(y)|≤L|(x−y)|##
this is the definition of lipschitz equation,
while
##|f(x)−f(x0)|≤f′+|(x−x0)|##
is from lagrange's theorem
 
Felafel said:
##|f(x)−f(y)|≤L|(x−y)|##
this is the definition of lipschitz equation,
while
##|f(x)−f(x0)|≤f′+|(x−x0)|##
is from lagrange's theorem

What does Lagrange's theorem say? Are you really allowed to apply it in this case? Are all the conditions satisfied?
 
uhm.. ok, i guess i don't have all the conditions to apply lagrange actually.
any hint about how else i can solve it?
 
Felafel said:
uhm.. ok, i guess i don't have all the conditions to apply lagrange actually.
any hint about how else i can solve it?

Did you prove already that every differentiable functions is continuous? Can you try to adapt that proof?
 
Question: A clock's minute hand has length 4 and its hour hand has length 3. What is the distance between the tips at the moment when it is increasing most rapidly?(Putnam Exam Question) Answer: Making assumption that both the hands moves at constant angular velocities, the answer is ## \sqrt{7} .## But don't you think this assumption is somewhat doubtful and wrong?

Similar threads

  • · Replies 13 ·
Replies
13
Views
2K
Replies
5
Views
2K
  • · Replies 1 ·
Replies
1
Views
2K
  • · Replies 4 ·
Replies
4
Views
2K
Replies
3
Views
2K
  • · Replies 8 ·
Replies
8
Views
2K
Replies
1
Views
1K
  • · Replies 0 ·
Replies
0
Views
367
  • · Replies 12 ·
Replies
12
Views
2K
  • · Replies 6 ·
Replies
6
Views
2K